A good problem

Discussion in 'Physics & Math' started by §outh§tar, Sep 7, 2008.

  1. §outh§tar is feeling caustic Registered Senior Member

    Messages:
    4,832
    A clock's minute hand has length 4 and its hour hand length 3. What is the distance between the tips at the moment when the distance separating them is increasing most rapidly?

    Don't cheat; I'm looking for thorough explanations.
     
  2. Google AdSense Guest Advertisement



    to hide all adverts.
  3. Steve100 O͓͍̯̬̯̙͈̟̥̳̩͒̆̿ͬ̑̀̓̿͋ͬ ̙̳ͅ ̫̪̳͔O Valued Senior Member

    Messages:
    2,346
    The change in distance will be most rapid at 90 degrees I think.

    So we get (3^2+4^2)^-2 = 5
     
  4. Google AdSense Guest Advertisement



    to hide all adverts.
  5. Vern Registered Senior Member

    Messages:
    695
    When the distance between them is one; they are pointing in the same direction.

    Nope that's wrong. It would be when the minute hand is approaching the hour hand and the right triangle formed by the tip distance is at 45 degrees.

    Not as easy as it first looks.
     
    Last edited: Sep 7, 2008
  6. Google AdSense Guest Advertisement



    to hide all adverts.
  7. Vern Registered Senior Member

    Messages:
    695
    And that would 1.414

    The square root of 2.
     
  8. AlphaNumeric Fully ionized Registered Senior Member

    Messages:
    6,702
    Why not just work in either polar or cartesian coordinates, work out the distance between the tips, say \(D\) in terms of the angle between them, then work out \(\partial_{\theta}D\). Then you have the expression for the relative velocity of the tips in terms of their angle. Then it's just a matter of finding when that function is extremised.

    This is basically M2 mechanics, ie working with vectors which are a function of something you then need to differentiate, so the first step is to draw a picture.
     
  9. RJBeery Natural Philosopher Valued Senior Member

    Messages:
    4,222
    After looking at the fire and the bucket of water, AlphaNumeric exclaims "A solution exists," and goes back to sleep...

    Please Register or Log in to view the hidden image!

     
  10. Vern Registered Senior Member

    Messages:
    695
    Hi AlphaNumeric; just from visualization it seems to me that the maximum velocity would occur when the tips form a right triangle with two sides equal to one. There's two places where that happens; one when the hour hand is approaching and one when it is departing. I'm not able to determine which of those would provide the greatest velocity. I just guessed it would be when minute hand is approaching the hour hand.

    I think you could probably work it out though

    Please Register or Log in to view the hidden image!

     
  11. Vkothii Banned Banned

    Messages:
    3,674
    Try a phase diagram, with two variables which are the tips of each hand. The phase space is straightforward - it's isn't spherical but circular (easy-peasy)...? But it's vector arithmetic too.
     
  12. andbna Registered Senior Member

    Messages:
    316
    The distance between the tips can be found based on thier angles of seperation, and using the cosine law:
    D(theta)=sqrt(25-24cos(theta))

    differentiate with respect to theta, to find the rate of distance change yields:
    D(theta)=(25-24cos(theta))^(1/2)

    d(theta)=0.5(25-24cos(theta))^(-1/2) * 24sin(theta)

    Now punch that into a graphing calculator and you will find that the rate of change in ditance is actualy the smallest at 90 degrees (or 1 pi radians)! (edit: being awake, it is painfully obviouse that 90deg does not equal a pi radian.)

    Of course, we are looking for a maxima (d't=0), so time for another derrivative (this one will be ugly, and hence may be more prone to error):

    d'(theta)=-.25(25-24cos(theta))^(-3/2) * (24sin(theta))^2 + 0.5(25-24cos(theta))^(-1/2) * 24cos(theta)

    d'(theta)=0
    0=-.25(25-24cos(theta))^(-3/2) * (24sin(theta))^2 + 0.5(25-24cos(theta))^(-1/2) * 24cos(theta)

    We can limit our range to 0<t<pi, and solve. Im going to use a GDC to solve. If your feeling particularly board, or masochistic, or perhaps know some secret that I don't to making this easy, feel free to solve it with limited (or no) mechanical aid

    Please Register or Log in to view the hidden image!



    My GDC says: at theta=0.72273425 occurs the maximum rate of distance change.
    So, now we can sub that back into D(theta), and calculate the distance between the hands:
    D(0.72273425)=2.645751318
    Assuming units in inches, the distance the two hands of your clock are apart at the time when the distance is changing most rapidly is approx 2.6 inches.

    Now I think I got that generaly correct, but please point out any errors. Good night.
    -Andrew
     
    Last edited: Sep 8, 2008
  13. przyk squishy Valued Senior Member

    Messages:
    3,203
    Before I turn in: work in a (rotating) reference frame where the minute hand is stationary. Then the distance is increasing the fastest when the tip of the hour hand is moving directly away from the tip of the minute hand, or the hour hand is at right angles to the line connecting the tips of the two hands. So \(d^2 + 3^2 = 4^2\) and the distance is \(\sqrt{7} = 2.64575 \ldots\)
     
  14. §outh§tar is feeling caustic Registered Senior Member

    Messages:
    4,832
  15. rpenner Fully Wired Valued Senior Member

    Messages:
    4,833
    The distance between the tips of the hands can be easily written in terms of one variable:
    \(d^2 = \left( 4 \sin 12 \theta - 3 \sin \theta \right) ^ 2 + \left( 4 \cos 12 \theta - 3 cos \theta \right) ^ 2\) Then the naive procedure is to solve for the extremal rate of change of the distance: \({ { \partial ^ 2 d} \over { \partial \theta ^ 2} } = { { \partial ^ 2 } \over { \partial \theta ^ 2} } \sqrt{ \left( 4 \sin 12 \theta - 3 \sin \theta \right) ^ 2 + \left( 4 \cos 12 \theta - 3 cos \theta \right) ^ 2 } = 0\)

    But you save yourself some time by noting that the distance is periodic in \(\phi = 11 \theta\) (because as the hour hand goes around once in twelve hours, the minute hand laps it 11 times). After some manipulation we have: \(d^2 = 25 - 24 \cos \phi\) and \({ { \partial ^ 2 d} \over { \partial \phi ^ 2} } = 12 { { 25 \cos \phi - 18 - 6 \cos 2 \phi } \over { d^3 } } = 0\) which has solution \(d = \sqrt{7}\) and \(\cos 11 \theta = \cos \phi = \frac{3}{4}\) (phi is in the first quadrent).

    But we can learn more if we solve a more general case.
    If the minute hand (length a) goes around the clock k > 1 times in the time the hour hand (length b) goes around once then we have, for a > b:
    \(d^2 = a^2 + b^2 - 2 a b \cos ( k - 1) \theta\)
    and \(\phi = ( k - 1) \theta\)
    and \({ { \partial ^ 2 d} \over { \partial \phi ^ 2} } = a b { { ( a^2 + b^2 ) \cos \phi - \frac{3}{2} a b - \frac{a b}{2} \cos 2 \phi } \over { d ^ 3 } } = 0 \) which has solution \(d = \sqrt{a^2 - b^2}\) and \(\cos (k - 1) \theta = \cos \phi = \frac{b}{a}\) (phi is in the first quadrent).

    I would like to see przyk finish the post, although the geometric approach is tougher for a text web.
     
  16. przyk squishy Valued Senior Member

    Messages:
    3,203
    Okay, a more complete reply:

    First, we work in a coordinate system whose origin is fixed at the centre of rotation of the hands and which is "rotating with" the minute hand. The tip of the minute hand is fixed at the position vector \(\bar{r}_{0}\), and the tip of the hour hand is rotating anticlockwise (at 11 complete revolutions every 12 hours if I'm not mistaken, though all that's important is that the angular velocity is constant) with a time-dependent position vector \(\bar{r} = \bar{r}(t), \; r \, < \, r_0\) :

    Please Register or Log in to view the hidden image!



    Defining \(\bar{s} = \bar{r} - \bar{r}_{0}\), direct calculation yields:
    \(\frac{\mathrm{d}s}{\mathrm{d}t} = \bar{v} \cdot \bar{1}_{s}\)​
    with the unit vector \(\bar{1}_{s} \equiv \frac{\bar{s}}{s}\) and \(\bar{v} \equiv \frac{\mathrm{d}\bar{r}}{\mathrm{d}t}\). Since the module of \(\bar{v}\) is constant, \(\bar{v} \cdot \bar{1}_{s}\) is maximal when \(\bar{s}\) and \(\bar{v}\) are parallel (intuitively, when all of \(\bar{v}\) is contributing to increasing \(\bar{s}\) and no component is lost in perpendicular motion). \(\bar{r}\) is perpendicular to \(\bar{v}\) (circular motion) and therefore also to \(\bar{s}\) at this point, which justifies the use of Pythagoras' theorem in calculating the separation, yielding: \(s_{0} = \sqrt{{r_{0}}^{2} - r^{2}}\)
     

Share This Page